Mastery Question Mdf

August 13, 2017 | Author: shaaish | Category: Hepatitis B, Hepatitis, Heart, Metabolism, Diseases And Disorders
Share Embed Donate


Short Description

Download Mastery Question Mdf...

Description

* Question 3 Maud is a 70 year old who presents with acute peri-umbilical abdominal pain gradually increasing in intensity. She is vomiting profusely and develops watery diarrhoea with flecks of blood after an hour of pain. Examination of the abdomen reveals localised periumbilical tenderness with some rigidity. Rectal examination is normal. An irregular pulse is noted and an ECG is recorded (shown below).

The MOST LIKELY diagnosis is: a) Acute appendicitis b) Acute pancreatis c) Perforated peptic ulcer d) Biliary colic e) Mesenteric artery occlusion Ans.E The clinical presentation is typical of mesenteric artery occlusion. This occurs most commonly in patients with atrial fibrillation leading to embolism. The ECG shows atrial fibrillation. Arteriography will show the vascular occlusion. * Question 4 A 15 year old young man has sudden onset of severe pain in his right lower abdomen commencing 2 hours ago. He has vomited several times in the last hour. He is rolling on the bed, stating that the pain is going down into his groin. T 37.1 degrees Celcius, P 110min, BP 135/ 80. Abdomen - soft, no rebound. Tender right testicle. Your immediate management is: a) i/v fluids and antibiotics b) arrange urgent ultrasound examination c) i/v metoclopramide (maxolon) d) refer for emergency surgery e) arrange for intravenous pyelogram (IVP) Ans.D The sudden onset of severe pain in the lower abdomen, groin or scrotum, in a young male under 25 years, should be considered to be testicular torsion until proved otherwise. This is a surgical emergency, as infarction of the testis can occur quickly, and surgical exploration should be undertaken urgently.

This patient has no fever, nor tenderness of the epididymis to indicate epididymo-orchitis. Antibiotic treatment will not help. Colour doppler ultrasound may show increased blood flow in infection and the absence of flow in advanced torsion. However, these are not reliable findings, and the investigation would waste valuable time. The vomiting is related to the pain, and would be alleviated by appropriate analgesia. Metoclopramide is not an immediate priority. The clinical picture is highly suggestive of testicular torsion rather than renal colic, thus IVP is not the appropriate immediate management. * Question 5 In a 3 year old child with signs and symptoms suggestive of bacterial meningitis, which of the following is the BEST initial management? a) Erythromycin IV b) Gentamicin IV c) Ceftriaxone IV d) Phenoxymethylpenicillin oral e) Amoxycillin oral Ans.C If bacterial meningitis is suspected clinically it is vital to immediately administer an appropriate antibiotic prior to urgent transfer to hospital, as meningococcal meningitis may be rapidly fatal. The drug of choice would be benzylpenicillin 60mg/kg up to 3g IV or IM, or ceftriaxone 50mg/kg up to 2g IV in patients hypersensitive to penicillin or when further drug treatment may be delayed. * Question 14 A 15kg child with a known food allergy to peanuts suddenly develops anaphylaxis. The RECOMMENDED immediate management is: a) 0.1ml of Adrenaline 1:1000 by deep intramuscular injection b) 0.1ml of Adrenaline 1:10,000 by deep subcutaneous injection c) 0.15ml of Adrenaline 1:1000 by deep intramuscular injection d) 0.15ml of Adrenaline 1:1000 by subcutaneous injection e) 0.15ml of Adrenaline 1:10,000 by deep intramuscular injection Ans.C Adrenaline 1:1000 is recommended as it is readily available, and this concentration contains 1mg of adrenaline per ml. The recommended dose of 1:1000 adrenaline is 0.01mg/kg body weight by deep intramuscular injection, so a 15kg patient would require 0.15ml. * Question 18

A 50 year old woman has had major abdominal surgery yesterday. You are called to see her urgently as she has symptoms of shock. Which ONE of the following examination findings is of MOST concern? a) the patient is restless and confused b) Temperature 39.2 degrees Celsius c) pulse 130, sinus tachycardia d) urine output over past 4 hours of 120ml e) BP 80/45 mm Hg Ans.E The above signs taken together suggest a picture of shock, probably septic shock. Hypotension (defined as systolic BP 40mm Hg fall from baseline level) is a sinister development and requires urgent attention. It is often a LATE manifestation of circulatory failure. Thus it is the most alarming of these findings, and the one most indicative of the urgency of this situation. Question 20 Aidan, a 3 month old boy, presents with paroxysms of coughing associated with cyanosis, lethargy and poor feeding for several days. On examination, between episodes of coughing, he is afebrile and examination is normal. What is the NEXT step in management? a) Admission to hospital b) Nasopharyngeal aspirate and review in 24 hours c) Immunisation at this visit with DTP and review in 24 hours d) Erythromycin syrup and review in 24 hours e) Trial of salbutamol by mask Ans.A The clinical picture suggests respiratory infection with Bordetella pertussis. The history of cyanosis associated with the coughing suggests the need for admission, but in addition, children under 6 months of age usually require hospital admission for pertussis because of the risk of complications. Complications include respiratory arrest, bacterial pneumonia and encephalitis. Salbutamol has not been shown to be helpful in a child of this age and is of no benefit. Immunisation at a later date should be encouraged even if the child has had pertussis. Erythromycin is not curative but may reduce infectivity. Question 34

Mary is an attendant at a local accommodation centre and has an intensely itchy rash on her wrists and arms that has been present for the past few days (see figure).

She has recently bought a new watch and wonders if this is the cause of the problem. The MOST LIKELY diagnosis is: a) Papular urticaria b) Tinea c) Contact dermatitis d) Eczema e) Scabies Ans.E Scabies is a skin infestation by the mite Sarcoptes scabiei. It is generally spread by skin to skin contact such as in crowded areas, poverty, sexual contact and casual contact. The mite can live for 2 days outside the human body, so infection by contact with bed linen and other infected material is possible. Intense itch is characteristic of the condition - if it is not itchy, it is not scabies. Distinct erythemato-papular itchy nodules are due to an allergic reaction to the mite, its faeces and its larvae. 0.5-1.0cm "burrows" can often be found on the fingers and wrist. Contact dermatitis to her watch would produce a local contact dermatitis. Question 36 The MOST appropriate treatment for first degree atrioventricular (AV) heart block is: a) An artificial pacemaker b) Isoprenaline hydrochloride (Isuprel) c) Atropine d) Digoxin e) Requires no treatment

Ans.E First degree AV block often does not require any treatment. Acute treatment of extreme bradycardia or second degree AV block (Mobitz type II) may require atropine or isoprenaline, but temporary pacing is the preferred treatment. Permanent pacing is recommended for distal block (Mobitz type 2) because of frequent early progression to third-degree atrioventricular block. Most patients with third degree (complete) AV block will require permanent cardiac pacing. Drugs such as digoxin may be the cause of an AV block and need to be ceased or the dose reduced. Question 13 Which of the following statements about patent ductus arteriosus is INCORRECT? a) It occurs frequently as an isolated phenomenon b) Cyanosis is usually present c) It causes a pansystolic 'machinery' murmur at the upper left sternal edge d) There is a wide pulse pressure e) Treatment is by surgical closure

Ans.B Cyanosis is not usually present unless a right to left shunt develops. Patent ductus arteriosus is usually an isolated problem occurring most commonly in females. There are often no symptoms until later in life, when heart failure or infectious endocarditis develops. Clinical signs include a continuous murmur and a bounding peripheral pulse with wide pulse pressure due to shunting of blood from the aorta to the pulmonary artery. Question 16 Which of the following features is UNLIKELY to be due to arterial ischaemia? a) Pain along the buttock and thigh after exertion b) Weakness of the buttock and thigh c) Shooting pain from buttock along the back of the leg to calf d) Weakness of the leg e) Smooth shiny skin on the leg below the knees Ans.C Diffuse pain, weakness and paralysis are all signs of arterial ischaemia. Characteristically the pain is a cramp-like ache due to the release of paininducing metabolites in muscle. Due to the aetiology, the pain is diffuse and cannot be localised, as can the shooting pain of nerve irritation.

Question 19 Harold, aged 24 years, presents with fatigue, shortness of breath on exercise and orthopnoea. On examination there are signs of moderate left-sided heart failure. A grade III pansystolic murmur is heard most prominently at the apex and radiating into the left axilla. Which of the following conditions is the MOST LIKELY diagnosis? a) Mitral stenosis b) Mitral regurgitation c) Aortic stenosis d) Aortic regurgitation e) Tricuspid stenosis Ans.B Mitral regurgitation presents as fatigue, exertional dyspnoea and orthopnoea. It is associated with a pansystolic murmur loudest at the apex but radiating over the praecordium and into the axilla. It may also be associated with a short mid-diastolic flow murmur following a third heart sound, due to the rapid flow of blood into the dilated left ventricle. The second heart sound is normal. Question 23 Victor, a 36 year old man, has known ischaemic heart disease. He complains of a recent increase in frequency of chest pain and presents with a prolonged episode of chest pain. Management includes all of the following EXCEPT: a) b) c) d) e)

Admission to hospital Plasma troponin Continuous ECG monitoring Thrombolytic therapy and oral aspirin Continuous IV nitroglycerin infusion

Ans.E Clinically this patient has unstable angina. Management should include continuous ECG monitoring, admission to hospital and plasma troponin to exclude myocardial infarction. Continuous IV nitroglycerin infusion should only be used when required. If there is no improvement in 24-48 hours, cardiac catheterisation and angioplasty are indicated. uestion 27 Malcolm, a 55 year old man, presented with symptoms of gastro-oesophageal reflux disorder (GORD). You referred him for a gastroscopy which has not

revealed any abnormality. He still complains of bloating and heartburn. Which of the following would you advise? a) Reflux has been excluded as a cause of his symptoms b) A trial of medication is inappropriate c) Endoscopy detects the presence of reflux in only 60-80% of patients d) It is important to treat his symptoms with a trial of medication e) He should have a repeat endoscopy in 6 months Ans.D Some patients have symptoms of gastroesophageal reflux disease but do not have endoscopic evidence of reflux or oesophagitis. A trial of medication is the treatment of choice as many patients will still respond to this.

Question 32 Herman is a 57 year old man who is recovering from a hitherto uncomplicated myocardial infarction. On the fourth day he complains of sudden onset of palpitations. Initial examination confirms a tachycardia with blood pressure of 140/80. The ECG shows the following rhythm (see figure).

The first line treatment for this patient is: a) Carotid sinus massage b) Digoxin IV c) Verapamil IV d) Lignocaine IV e) DC cardioversion P wave duration ≤ 0.12 s PR interval 0.12–0.22 s QRS complex duration ≤ 0.10 s Corrected QT (QTc) ≤ 0.44 s in males ≤ 0.46 s in female QTcB = QT/√2(R − R) Bazett’s square root formula QTcF = QT/√3(R − R) Fridericia’s cube root formula

Ans.D This ECG shows ventricular tachycardia with a rate of 150 b.p.m. There is a rapid ventricular rhythm with broad, abnormal QRS complexes. Since his blood pressure is well maintained, medical treatment is indicated as first line approach. Lignocaine IV or sotalol IV or amiodarone IV can be used. DC cardioversion is required if medical therapy is unsuccessful. If the cardiac output and blood pressure are very depressed, emergency DC cardioversion must be considered. Question 37 Edith is a 70 year old woman who presents with palpitations. Her ECG is shown below.

What is the diagnosis? a) Atrial flutter b) Atrial fibrillation c) Atrial premature beats d) Sinus arrhythmia e) 1st degree AV block Ans.B This ECG shows atrial fibrillation. There are no p waves and the rhythm is irregularly irregular which causes the patient to perceive palpitations. Question 4 Clarice, 26 years, presents to you concerned because she has noticed that a dark mole on her thigh has become enlarged, slightly lumpy and itchy over the last two months. You suspect it may be a malignant melanoma. The MOST APPROPRIATE initial management would be to: a) Ask Clarice to return for review in three months b) Take a incisional biopsy of the lesion for histopathology c) Treat the lesion using liquid nitrogen d) Remove the lesion using laser e) Undertake an elliptical excision clear of the margin for histopathology Ans E If a malignant melanoma is suspected then an accurate pathological report is required to guide further management. For this reason it is important that the initial management involves complete removal of the lesion

without destruction of the tissue. Early detection and removal of melanomas leads to better outcomes (Clark's level one and two melanomas have a five year prognosis of >90%). If a melanoma is diagnosed then referral to a plastic surgeon is necessary for a wide local excision involving a margin of 1-3 cm and to a depth of the deep fascia. Question 6 Benny has always loved to go clubbing, and often after a few drinks at the end of a night of dancing, he ends up having casual sex with someone he meets at the nightclub. Benny had his first hepatitis B serology testing done last week. These are his test results: HBsAg = positive HBsAb = negative IgM HBcAb = positive HBeAg = positive. What is the MOST LIKELY cause of these results? a) Benny has been vaccinated in the past for hepatitis B and is now immune b) Benny has had hepatitis B infection sometime in the past and it has resolved, leaving him with life-long immunity c) Benny is a hepatitis B carrier d) Benny has acute or current hepatitis B infection e) Benny has early liver cirrhosis Ans.D Benny is HBsAg positive which occurs 1-6 months after exposure to the hepatitis B virus and indicates acute infection. If HBsAg persists after 6 months, it defines carrirer status. HBsAb is not present (it would be positive following vaccination). IgM HBcAb is present in acute infection only (IgG HBcAb is present in highly infective carriers and in acute infection). HBeAg is present and implies high infectivity in recent infection and carriers. Benny needs education about hepatitis B, safe sex & drug use * Question 7 The clinical features associated with raised intracranial pressure include all of the following EXCEPT: a) morning headache b) vomiting c) presence of papilloedema d) decrease in conscious state e) falling blood pressure with a falling pulse

Ans.E Rising blood pressure (not falling) in combination with a falling pulse rate is a classical feature of rising intracranial pressure known as the Cushing response. Headache occurs as a result of the deformation of intracranial blood vessels and dural membranes which arises from conditions which give rise to raised intracranial pressure. The headache is worst in the morning (as is vomiting) and is aggravated by coughing, sneezing or stooping. When present papilloedema (swelling of the nerve fibres of the optic disc) is highly suggestive of raised intracranial pressure. A decrease in conscious state commencing with confusion and progressing through various grades of coma is also seen with increasing intracranial pressure. Question 9 Which of the following is FALSE regarding neural tube defects and folate before and during pregnancy? a) Folate intake should be increased at least one month before and three months after conception b) Most women before and during pregnancy need 0.5mg folate daily c) Women on anti-epileptic medication may require 5mg folate daily before and during pregnancy d) Folate reduces the incidence of neural tube defects which occur at the rate or 1:5000 pregnancies e) Women with a family history of neural tube defects need more folate before and during pregnancy Ans.D Pregnant women are at increased risk of folate deficiency due to the high demand of the developing foetus. Deficiency in the first few weeks of pregnancy can cause neural tube defects in the newborns. Neural tube defects occur at a rate of 1:500 pregnancies. The other options are true. * Question 10 Pamela aged 45 years, attends having found a lump in the upper outer quadrant of her right breast two days ago. She is concerned about the likelihood of cancer. In order to diagnose the nature of the lump you invoke the use of the "triple test" or "triple assessment". The triple test consists of: a) Clinical examination, mammography, magnetic resonance imaging (MRI) b) Mammography, ultrasound, fine needle biopsy c) Clinical examination, mammography, fine needle biopsy

d) Clinical examination, ultrasound, magnetic resonance imaging (MRI) e) Ultrasound, fine needle biopsy, magnetic resonance imaging (MRI) Ans.C Management of breast lumps is now based on the triple test, which combines the results of clinical examination, mammography (+/- ultrasound) and fine needle aspiration biopsy. When combined, these tests give a sensitivity of 95-99% in the diagnosis of breast lumps.

Question 22 Beth, aged 6 months, is brought to see you by her mother who has noticed her eyes are not always lined up. You are concerned Beth may have a squint (strabismus). Which of the following statements regarding strabismus is CORRECT? a) Investigation is unnecessary in this age group as strabismus improves with time b) By the age of 6 months Beth's eyes should be constantly well aligned c) Strabismus is rarely a marker of other ocular disease d) Strabismus is not associated with amblyopia e) The corneal light reflex is a reliable test to diagnose strabismus Ans B A baby's eyes should be constantly well aligned by the age of 5 to 6 months. Intermittent ocular deviation should be investigated if present at six months, as it may be a marker of severe underlying ocular or neurologic disease. It should never be assumed that the strabismus will be outgrown. The corneal light reflex test should not be relied upon to diagnose or exclude strabismus. The cover test is a more accurate diagnostic test. Strabismus may lead to amblyopia, which in turn may result in permanent loss of vision if it is not corrected by 4 to 6 years of age. Question 25 Robyn, aged 43 years, is known to have gallstones. On this occasion she presents with the acute onset of severe pain which was at first central in location but has now moved to the right costal margin and radiates to the back. She is pyrexic, slightly tachycardic and has tenderness over the area of the gall bladder but no rigidity of the abdomen. The MOST APPROPRIATE MANAGEMENT would be to: a) Allow her to return home and review her in two to three days allowing time for the attack to settle

b) Admit her to hospital and treat her with IV fluids and antibiotics c) Refrain from use of opiates due to risk of addiction d) Admit her to hospital for immediate surgery e) Aim to delay the operation for several weeks to months Ans B Robyn has acute cholecystitis. Initial management includes IV fluids and nil by mouth, pain relief with parenteral opiate administration and a short, intensive course of antibiotics. The patient is monitored and immediate operation is ONLY indicated if the fever does not settle or symptoms worsen. Immediate operation is not warranted, as there is no indication of perforation of the gall bladder or peritonitis. However, early operation for acute cholecystitis is now recommended compared to delaying surgery.

* Question 26 Kari is 7 months old and has not received any immunisations. She presents with two weeks of paroxysmal coughing and vomiting, but is relatively happy between paroxysms. You suspect she may have whooping cough (pertussis). Kari lives at home with her mother, father and older brothers, aged 2 and 4 years. Neither of her brothers have been immunised against pertussis. Choose the BEST INITIAL MANAGEMENT option from the list below. a) Arrange to have Kari admitted to hospital and isolated immediately b) Report the family to the child protection agency in your state for failing to immunise their children c) Vaccinate Kari immediately with DTPa-hepB or DTPa d) Prescribe oral erythromycin for Kari and the whole family e) Take a nasopharangeal aspirate for diagnosis, and await confirmation of diagnosis prior to starting any other treatment measures Ans D Whilst it is important to obtain a laboratory diagnosis of pertussis, this should not delay treatment, which should be commenced after appropriate nasopharangeal aspirate or serological samples are collected. Kari should be treated with erythromycin 10mg/kg/dose up to 250mg orally 6 hourly for 10 days, as should all household and other close contacts. Hospitalisation and isolation are unnecessary unless the clinical condition of the patients warrants inpatient management. Catch-up vaccination should be addressed, but is not the most immediate concern here. There is no requirement to report the family to authorities if they are conscientious objectors to immunisation.

Question 30 Esther is 7 years old. She presents with a large yellow crusted lesion on her left cheek and similar yellow crusted lesions along her left lower jawline. She has no lesions or rash elsewhere and is otherwise well. Which is the MOST ACCURATE statement regarding this condition? a) Herpes simplex is the likely causative organism b) It is important not to disturb the crusts c) Esther should be screened for immune deficiency d) Topical mupirocin is an appropriate treatment e) Oral antibiotics should be commenced as early as possible to prevent septicaemia developing Ans D The most likely diagnosis is impetigo, with the ruptured vesicles that form yellow crusts and weeping erosions being quite typical of the lesions. Herpes simplex has a different clinical presentation. In childhood, primary HSV infection usually presents as severe acute gingivostomatitis. Impetigo is a very common, highly contagious infection, and does not suggest an underlying immune deficiency. The usual pathogen is Staphylococcus aureus, or Streptococcus pyogenes. For mild or localised impetigo, topical mupirocin 2% ointment or cream 3 times daily for 7 days is appropriate treatment. Whilst oral antibiotics may be indicated for more widespread infection, septicaemia is not a usual sequelae of this common condition. Bathing the lesions to remove the crusts may be helpful. Question 32 One minute after birth an infant shows deep cyanosis of the trunk and limbs, makes no reaction to a catheter inserted into the nose, is limp but takes an occasional gasp. What is the Apgar score? a) 0 b) 1 c) 2 d) 3 e) Insufficient data Ans E The table below shows the data required to determine an Apgar score. The scenario given lacks information about the heart rate. Other data given are compatible with a score of 0. Apgar Score 0 1 2 Sign

Heart rate

absent

100 beats/min

Respiratory effort

absent

irregular, weak cry

regular, strong cry

Muscle tone

flaccid

some flexion of upper well flexed, active extremities motion

Reflex irritabilities

no response

grimace

cough or sneeze

Colour

central cyanosis

peripheral cyanosis

completely pink

Question 40 Lulu is a three year old child who has swallowed kerosene and is brought immediately to the hospital casualty department. Which of the following measures should be undertaken in the immediate management of Lulu's problem? a) Gastric lavage b) An emetic c) Chest x-ray d) Intravenous saline e) Methicillin

Ans C Kerosene is an aliphatic, highly volatile hydrocarbon which is poorly absorbed from the gastrointestinal tract. Pneumonitis through aspiration of fumes is the predominant toxic mechanism in children and respiratory distress can be severe and occur rapidly. While a chest xray is not useful for the prediction of lung involvement, serial chest xrays are important to monitor progression. In children who present with lethargy, fever or respiratory signs in the first 4 hours 80% develop pneumonitis. Gastrointestinal irritation is common with nausea and vomiting. There may also be a high fever within 30 minutes of ingestion. Management should be conservative and decontamination (emesis or gastric lavage) should not be attempted - it merely increases the risk of aspiration, and development of pneumonitis. IV saline may be required if haemolysis from the kerosene occurs and hypotension develops. Question 4 Leigh is a 60 year old woman who has been hypertensive for 5 years. Her BP now is 160/115 mm Hg. Recently she has been getting increasingly short of

breath. Clinical assessment confirms congestive cardiac failure. Which of the following drugs would be preferred for management? a) Propranolol b) Verapamil c) Diltiazem d) Lisinopril e) Felodipine D. Linisopril is an angiotensin-converting enzyme inhibitor. This is the treatment of choice, as it lowers systemic vascular resistance and venous pressure and reduces the levels of circulating catecholamines, thus improving myocardial performance. It is important to observe for first-dose hypotension. Calcium channel blockers (e.g. verapamil, diltiazem) may have a detrimental effect on left ventricular function in patients with heart failure. At present, there is no general agreement on the timing of beta blocker (propanolol) therapy. It is currently reserved for those patients who remain symptomatic whilst on maximal therapy with other agents. * Question 5 In which of the following situations would a barium swallow be preferable to an endoscopy as a FIRST LINE investigation? a) Patient complains of coughing after meals b) Patient complains of difficulty swallowing c) Patient with nocturnal symptoms only d) Patient with bloating after meals e) Patient has water-brash B. Difficulty swallowing (or dysphagia) is a functional problem and a barium swallow is preferable to an endoscopy in this instance. Observations on the barium swallow may suggest oropharyngeal or cricopharyngeal dysfunction (including misdirection of barium into the trachea or nasopharynx), prominence of the cricopharyngeal muscle, a Zenker's diverticulum or a narrow pharyngeo-oesophageal segment. Disordered oesophageal motility or structural abnormalities such as small diverticula, webs, and minimal extrinsic impressions of the oesophagus may be recognised only with motionrecording techniques. Question 13 A healthy six year old child without cyanosis or dyspnoea on exercise is examined for migration to Australia. His pulse is 84 per minute, B.P. 100/60, radial pulse and jugular venous pressure normal and there is no evidence of cardiomegaly. On auscultation in the 2nd left intercostal space the 1st and 2nd

heart sounds are audible with fixed splitting of the 2nd heart sound and a midsystolic pulmonary ejection murmur is heard. The MOST likely diagnosis is: a) Pulmonary stenosis b) Atrial septal defect (ASD) c) Innocent pulmonary ejection murmur d) Ventricular septal defect (VSD) e) Patent ductus arteriosus (PDA)

B. In an asymptomatic patient an ASD is often diagnosed as a loud P2 with fixed splitting and an ejection murmur heard in the pulmonary area due to increased blood flow to the right heart. A VSD large enough to produce these signs would be symptomatic and usually would cause cardiomegaly. Innocent pulmonary ejection murmurs do not cause fixed splitting of P2, and a PDA causes a continuous murmur. In pulmonary stenosis P2 is often soft or inaudible and the JVP is usually elevated. Question 23 Belinda, aged 44 years, presents complaining of heavy, prolonged periods (menorrhagia) and severe period pain (dysmenorrhoea) that has gradually become worse during the past year. Her periods are still quite regular. Which of the following possible causes is UNLIKELY? a) Adenomyosis b) Endometriosis c) Uterine cancer d) Fibromyoma e) Ovarian failure E. Ovarian failure presents as irregularity and scarcity of menstruation, rather than menorrhagia and dysmenorrhoea. Adenomysosis, endometriosis, uterine cancer and fibromyoma are all possible causes of menorrhagia and secondary dysmenorrhoea. Other causes include uterine myomas and polyps, intrauterine contraceptive devices and congenital malformations (eg. bicornuate and septate uterus). Question 31 In a child with chickenpox which of the following drugs is MOST LIKELY to cause Reye's syndrome? a) Paracetamol b) Aspirin

c) Codeine d) Penicillin e) Prednisolone B. Reyes Syndrome involves acute encephalopathy and fatty infiltration of the liver following an acute viral infection, including influenza and varicella. Foreign chemicals, especially salicylates (including aspirin), and intrinsic metabolic defects have also been implicated. The use of salicylates (eg aspirin) during an acute viral illness such as chicken pox (varicella) increases the risk of Reyes syndrome by as much as 35-fold. Question 33 Sarah is an 8 week old girl who has persistent regurgitation. Which of the following features suggests the need for further investigation? a) Sarah is underweight for her age b) She regurgitates after every meal c) She has episodes of uncontrollable crying d) She arches her back on occasion and stops feeding e) Sarah was born 2 weeks premature A. Regurgitation after every meal suggests gastroesophageal reflux, but of itself is not a worrying feature. Underweight for age, however, suggests failure to thrive and needs investigation. Unsettled and irritable behaviour is very common in the first 6-12 weeks of life. In isolation it is not a concern. Question 36 Sue, a three year old girl, presents with shortness of breath and wheeze that have developed over the last two days. Examination reveals an afebrile, moderately tachypnoeic child with widespread scattered wheezes on auscultation. You decide to administer a bronchodilator (salbutamol). The recommended method of delivery of salbutamol for Sue is: a) Syrup b) Nebuliser c) Breath activated inhaler d) Metered dose inhaler with a spacer e) Metered dose inhaler with a spacer and face mask

E. For the treatment of acute asthma in a child 6 years of age, a large volume spacer may be substituted. Salbutamol administered via these routes has been shown to be equally effective to nebulised salbutamol. Six puffs of salbutamol via MDI and spacer is the equivalent of a 2.5 mg nebule, while 12 puffs equals a 5mg nebule. Question 38 In iritis (uveitis) the pupil is: a) Eccentric and reacts briskly to light b) Concentric dilated and reacts briskly to light c) Eccentric and reacts sluggishly to light d) Concentric, constricted and reacts sluggishly to light e) Concentric, dilated and reacts sluggishly to light

D. In iritis the pupil is concentric, constricted and sluggish. The signs of acute anterior iritis are pupillary miosis and perilimbal flush. The ciliary body constricts due to irritation and therefore is less able to respond quickly to light. Question 40 Trevor, a male infant weighing 2.4 kg at birth after a normal labour, becomes jaundiced at 12 hours of age. Which of the following conditions would be the MOST LIKELY cause of the jaundice? a) Gram negative septicaemia b) Jaundice of prematurity c) Biliary atresia d) Physiological jaundice e) Rh incompatibility

E. Jaundice appearing in the first 24 hours of life is most commonly due to haemolytic disease of the newborn due to incompatibility to Rh, ABO or one of the other rare antigens. Other causes of early jaundice include transplacental infections such as CMV, toxoplasmosis and rubella. Jaundice of prematurity and physiological jaundice occur between days 2-5. Biliary atresia presents with jaundice after the first week of life. Question 3 Which of the following criteria enable a clear distinction to be made between haemorrhage and thrombosis in a patient with a cerebrovascular accident?

a) b) c) d) e)

The progress of the clinical features The degree of loss of consciousness The abruptness of onset The presence or absence of headache None of the above

E. Intracerebral haemorrhages tend to be dramatic and accompanied by a severe headache. However, there really is no clinical way of reliably distinguishing between an intracerebral haemorrhage and a thromboembolic infarction, as both produce a sudden focal deficit.

* Question 4 Bel is 20 years old and has had one allergic reaction to a bee sting. She states that there is a family history of bee sting allergy . Which of the following is NOT useful advice for Bel? a) Do not drink out of an open soft drink can that has been left outdoors b) Have a supply of antihistamines on hand c) Insect repellents are useful to prevent bee stings d) Do not walk barefoot around swimming pools e) Always carry an adrenalin 1:1000 injection, e.g. EpiPen, and know how to use it

C. Insect repellents have not been shown to be useful in preventing bites from stinging insects. Anyone with a known allergy to stinging insects should know how to administer adrenalin 1:1000 subcutaneously and have it with them at all times. EPIpen is a commercial preparation which is supplied with an autoinjection device. Avoiding behaviours likely to lead to a sting - such as those mentioned in the options and avoiding colourful clothes and perfumes which attract insects - is also important. * Question 5 John is a 30 year old professional athlete who suddenly develops persistent dull upper left chest pain which is not related to exertion. Although not related to respiration, it causes mild restriction in breathing. There were no related respiratory or cardiac symptoms; he is not distressed and is afebrile. Which of the following diagnoses is LEAST likely? a) Spontaneous pneumothorax b) Functional chest pain (anxiety neurosis) c) Costo-chondral syndrome d) Muscle strain e) Pleurodynia ( Bornholm's disease)

E. Bornholm's disease is due to an infection by Coxackie B virus. It is often associated with an acute upper respiratory tract infection with fever, pleuritic chest pain and upper abdominal pain. These pains can be severe and associated with tachypnoea. A spontaneous pneumothorax, functional chest pain, costochondritis or acute muscular strain would be more likely in this patient. Question 9 Abdul is a 58 year old man who presents with transient episodes of vertigo, slurred speech, diplopia, and paresthesia. Which of the following is the MOST likely diagnosis? a) Basilar artery insufficiency b) Anterior communicating artery aneurysm c) Hypertensive encephalopathy

d) Pseudobulbar palsy e) Occlusion of the middle cerebral artery Bulbar palsy Synonyms: lower motor neurone dysarthria, neuromuscular dysarthria, atrophic bulbar paralysis. Bulbar relates to the medulla. Bulbar palsy is the result of diseases affecting the lower cranial nerves (VII - XII). A speech deficit occurs due to paralysis or weakness of the muscles of articulation which are supplied by these cranial nerves. The causes of this are broadly divided into:

1)Muscle disorders.

2)Diseases of the motor nuclei in the medulla and lower pons. 3)Diseases of the intramedullary nerves of the spinal cord. 4)Diseases of the peripheral nerves supplying the muscles. • Lips - tremulous • Tongue - weak and wasted and sits in the mouth with fasciculations • Drooling - as saliva collects in the mouth and patient is unable to swallow (dysphagia) • Palatal movements are absent • Dysphonia - rasping tone due to vocal cord paralysis; nasal tone if bilateral palatal paralysis • Articulation - difficulty pronouncing r; unable to pronounce consonants as dysarthria progresses Causes: • • • • • • • • •

Diphtheria Poliomyelitis Motor neurone disease e.g. progressive bulbar palsy (features of pseudobulbar palsy may also be present) Syringobulbia Cerebrovascular events of the brainstem Brainstem tumours After radiotherapy for nasopharyngeal carcinoma After surgery for acoustic neuroma Guillain-Barré syndrome

Pseudobulbar palsy Synonyms: upper motor neurone dysarthria, spastic dysarthria. Pseudobulbar palsy results from disease of the corticobulbar tracts. Bilateral tract damage must occur for clinically evident disease as the muscles are bilaterally innervated. • unable • • • • • • • •

Tongue - paralysed, no wasting initially and no fasciculations; "Donald duck" speech; to protrude Palatal movements absent Dribbling persistently Facial muscles - may also be paralysed Reflexes - exaggerated e.g. jaw jerk Nasal regurgitation may be present Dysphonic Dysphagic Emotional lability may also be present

There may also be neurological deficits in the limbs e.g. increased tone, enhanced reflexes and weakness. Causes: • • • • • •

Cerebrovascular events e.g. bilateral internal capsule infarcts Demyelinating disorders e.g. multiple sclerosis Motor neurone disease High brainstem tumours Head injury Neurosyphilis

A. Transient ischaemic attacks involving the posterior brain circulation, i.e. the basilar artery, are characterised by diplopia, vertigo, vomiting, dysarthria, ataxia and hemisensory loss. Question 28 The FIRST sign of salicylate poisoning in children is usually: a) Delirium b) Coma c) Hyperventilation d) Hyperpyrexia e) Convulsions •

The following 4 categories are helpful for assessing the potential severity and morbidity of an acute, single event, nonenteric-coated, salicylate ingestion: o Less than 150 mg/kg - Spectrum ranges from no toxicity to mild toxicity o From 150-300 mg/kg - Mild-to-moderate toxicity o From 301-500 mg/kg - Serious toxicity o Greater than 500 mg/kg - Potentially lethal toxicity

Pathophysiology

The toxic effects of salicylates are complex. Respiratory centers are directly stimulated. Salicylates cause an inhibition of the citric acid cycle and an uncoupling of oxidative phosphorylation. In addition, lipid metabolism is stimulated, while amino acid metabolism is inhibited. Catabolism occurs secondary to the inhibition of ATP-dependent reactions with the following results: • • • • •

Increased oxygen consumption Increased carbon dioxide production Accelerated activity of the glycolytic and lipolytic pathways Depletion of hepatic glycogen Hyperpyrexia

Acid-base disturbances vary with age and severity of the intoxication. Initially, a respiratory alkalosis develops secondary to direct stimulation of the respiratory centers. This may be the only consequence of mild salicylism. The kidneys excrete potassium, sodium, and bicarbonate, resulting in alkaline urine. Metabolic effects A severe metabolic (ketolactic) acidosis with compensatory respiratory alkalosis may develop with severe salicylate intoxication. A paradoxical aciduria (hydrogen ion excretion) occurs with the depletion of sodium bicarbonate and potassium. Infants rarely present with a pure respiratory alkalosis. Respiratory alkalosis with a compensatory (high anion gap) metabolic acidosis defines the next stage in moderate-to-

severe intoxication. Potassium moves from the intracellular space to the extracellular space. Excretion of hydrogen ions produces acidic urine.

C. Aspirin has a two-fold toxic effect. First, it inhibits oxidative phosphorylation leading to a metabolic acidosis. The increased hydrogen ion concentration of the extracellular fluid stimulates the respiratory centre of the brain to cause hyperventilation. This is the primary effect in children. Second, aspirin directly stimulates the respiratory centre to cause hyperventilation leading to a respiratory alkalosis. This phenomenon is seen mainly in adults. * Question 29 Charles is a 48 year old businessman who presents for a general check-up and mentions that he is experiencing occasional fluttering sensations in his chest. A routine electrocardiograph (see figure) is taken.

Your first line of management should be: a) Reassurance b) Digoxin c) Captopril d) Beta blockade e) Quinidine

A. The ECG shows Charles has premature atrial ectopic beats. Often these are asymptomatic. They may, however, be sensed as an irregularity or heaviness of the heart beat. Treatment is not normally required, unless the ectopic beats provoke more significant arrhythmias. In such a situation Betablockade may be effective. Question 32 Which of the following situations is a CONTRAINDICATION to immunisation with a live attenuated vaccine? a) Pregnancy b) Diarrhoea c) Mild acute febrile illness d) Current antibiotic therapy

e) Breast feeding A. Pregnancy is a valid contraindication to immunisation with a live attenuated vaccine eg oral polio infection. Exposure to HIV, other immunodeficiency states and immunosuppressant treatments are also contraindications. Diarrhoea, minor acute illnesses, antibiotic therapy and breast feeding are not valid contraindications. Question 34 An obese man, aged 60 years, is admitted unconscious with a diagnosis of cerebral thrombosis. The most important IMMEDIATE management is: a) Insertion of an indwelling catheter b) Commencement of anticoagulant therapy c) Physiotherapy to prevent hypostatic pneumonia d) Insertion of an intravenous drip to prevent dehydration e) Positioning him on alternate sides 2 hourly

A. An indwelling catheter allows monitoring of fluid status as well as allowing urinary drainage. IV fluid therapy is not urgent due to the potential to exacerbate brain swelling, in the acute phase. Anticoagulant therapy is of no value in treating a fully developed and completed CVA, and should be used only in transient ischaemic attacks or developing progressive thrombosis. The other measures of physiotherapy and nursing care should follow. * Question 35 Which of the following statements about simple febrile convulsions is CORRECT? a) It usually occurs between 6-8 years of age b) Prognosis is poor c) The risk of developing epilepsy is 10% d) The convulsions last less than 15 min e) Investigation with lumbar puncture and CT is essential

D. Simple febrile convulsions last less than 15 minutes. They usually occur between 3 months and 5 years, with most occurring between 17 and 23 months of age. There is no difference in IQ at age 7 years between children

who have had a febrile convulsion and their seizure- free siblings. The risk of developing epilepsy following a simple febrile convulsion is 0.9% at age 7 years. Question 37 Katie, a twelve year old school girl, collapses suddenly at school, and is transported by ambulance with dextrose drip (60/ml min.) inserted. On examination, dolls eye reflexes are present and she is not responding to painful stimulus. Her vital signs are as follows: Resp. rate 14/min Pulse rate 50/min Sa02 100 % B/P 180/110 What is the NEXT step of management? a) Arrange for urgent scan b) Stop her dextrose infusion and start a saline infusion c) Give steroids d) Intubate the patient e) Neurosurgical consultation C. Katie has raised intracranial pressure as indicated by the hypertensive response in the presence of bradycardia and coma. Glucocorticoid steroids (eg dexamethasone) are most effective in reducing raised intracranial pressure and should be given first before the other measures. Katie will need a neurosurgical consultation and an urgent CT scan or MRI to diagnose the cause of increased intracranial pressure. Intubation will be required if her airway becomes compromised. Mannitol (IV) and hyperventilation to an arterial pCO2 of 25-30 mmHg may also be useful in controlling raised intracranial pressure.

* Question 38 The defect in visual fields MOST commonly associated with a pituitary tumour is: a) Crossed homonymous hemianopia b) Central scotoma c) Bitemporal hemianopia d) Total blindness in one field e) Peripheral concentric constriction and enlargement of the blind spot

C. As a pituitary tumour extends upwards from the diaphragma sellae and compresses the optic chiasm, it classically causes superior quadrantic defects followed by bitemporal hemianopia. It can however cause any variety of visual field defects, including unilateral (or bilateral) field defects in all quadrants, due to the variable position of the chiasm above the pituitary. Question 40 Which of the following would be MOST helpful in distinguishing cerebral infarction from cerebral neoplasm? a) History of headache b) Hemiplegia c) Chronology of development d) Carotid bruit e) Focal abnormality on electroencephalogram C. Chronology of development is the most important factor in differentiating cerebral infarction from cerebral neoplasm. Cerebral infarction tends to be a simple, sudden event or a series of stepwise events within hours to days. In comparison, neoplasms tend to be preceded by symptoms such as headache, progressive cognitive decline, seizures and vomiting, and may feature steadily progressive neurological signs. The other options are all variably present in both conditions and are not diagnostic.

Question 1 Bill is 65 years old and has just been diagnosed with type 2 diabetes. He returns to discuss the condition. What would you tell him about diabetic retinopathy? a) He should see an ophthalmologist straight away b) As Type 2 diabetes has just been diagnosed he won't need a referral for 5 years c) Retinopathy is not a problem in Type 2 diabetes d) Retinopathy is a rare complication of diabetes e) None of the above A. At diagnosis, one in six patients with Type 2 diabetes has retinopathy. If untreated, this progresses to cause retinal scarring, contraction of the vitreous humour and retinal detachment. Eventually about 85% of all patients will

show signs of retinopathy. All patients with Type 2 diabetes should be screened for retinopathy at the time of diagnosis, and then at least every two years thereafter. Laser therapy is very effective and halves the risk of visual loss from diabetic retinopathy. Question 3 Brian, a long standing patient of your practice wants to discuss a friend of his who has just had his nail removed because of a melanoma. Which of the following statements about malignant melanoma is CORRECT? a) This is a very common condition b) It is rarely fatal c) Removal of the nail constitutes a cure d) Mean survival time post diagnosis is 12 months e) This condition does not metastasise D. Melanomas which occur on the palms, soles or nail bed are called acral melanomas and are quite rare. Because of their position, they are not found until quite late, and hence have a very poor prognosis. They spread locally and metastasise to regional lymph nodes. Biopsy with removal of the entire digit is the definitive treatment. Question 19 Blood-stained discharge from the nipple of a 45 year old woman is MOST LIKELY due to: a) Gynaecomastia b) Duct papilloma c) Paget's disease of the nipple d) Fibroadenoma e) None of the above B. Duct papilloma typically presents with a unilateral serosanguineous or bloody nipple discharge. Paget's disease of the nipple presents with a chronic eczematous eruption indicating an underlying malignancy. Gynaecomastia (benign enlargement of the male breast) and fibroadenoma (smooth round asymptomatic breast lump) are not associated with blood stained nipple discharge. Question 22

A 60 year old man presents with severe abdominal pain, shock, moderate abdominal rigidity and intense lower back pain. Which of the following diagnoses is MOST LIKELY? a) Acute retrocaecal appendicitis b) Leaking aortic aneurysm c) Renal colic d) Acute cholecystitis e) Collapse of L4 vertebral body B. A leaking abdominal aortic aneurysm typically presents with severe abdominal pain, shock, abdominal rigidity and intense lower back pain. It can be mistaken for renal colic, acute cholecystitis, and retrocaecal appendicitis. However circulatory shock is not usually present in these conditions. The BP may be increased due to pain. Collapse of the L4 vertebral body results in more localised pain without shock. Question 25 A patient who has been treated with a preparation containing horse serum develops urticaria followed by swelling of the tongue and dyspnoea. Which of the following is the MOST APPROPRIATE immediate treatment? a) Tracheotomy b) Subcutaneous adrenaline c) Intravenous hydrocortisone d) Intravenous promethazine (Phenergan) e) Oxygen therapy B. This is acute angio-oedema and there is a risk of upper airways closure, so subcutaneous adrenaline should be given first. Question 32 Which one of the following is NOT characteristic of an upper motor neurone lesion? a) Clonus b) Extensor plantar response c) Intact superficial reflexes d) Increased tone e) Hyperactive tendon reflexes

C. The following signs result from lesions in the motor system proximal to the alpha motor neurone: spasticity (hypertonia predominant in flexors of arms and extensors of legs which is of a clasp-knife nature); paralysis or weakness predominantly of extensors in arms and flexors in legs; hyperreflexia; extensor plantar response; clonus and Hoffmann's reflex. The extensor plantar response is an example of loss of a superficial reflex. Question 35 A 67 year old man presented three days after a stent was inserted for the treatment of persistent angina. He now complains of a persisting "different" chest pain and shortness of breath on exertion. On examination you find he is pale and slightly sweaty with: pulse rate 110 regular with pulsus paradoxus; BP 100/90; T 38.0 degrees Celsius; pedal oedema; bilateral basal crepitations in his chest; and a JVP elevated 3cm. Heart sounds are dual. The MOST LIKELY cause of his condition is: a) Hospital acquired pneumonia b) Post-infarct left ventricular remodelling with failure c) Extension of the infarct secondary to stent failure d) Pericarditis with tamponade e) Bacterial endocarditis

D. Pericarditis is a recognised, although uncommon, complication of invasive cardiac treatments. The patient can lose the pain of angina, only to have it replaced by a more vague chest discomfort. Inflammatory pericarditis results in an effusion which can rapidly escalate into tamponade. None of the other options would show signs of biventricular failure this quickly.

* Question 36 A 60 year old engineer was admitted to hospital because of fever, cough, and pleuritic chest pain. His temperature was 40 degrees Celsius. Physical examination and x-ray of the chest indicated right lower lobar pneumonia. Sputum smear and culture demonstrated pneumococci. The patient had a history of allergy to penicillin, and therefore tetracycline therapy was instituted. After several days, fever and leucocytosis decreased and x-ray of the chest showed some clearing of infiltrate. On the 7th hospital day, his temperature spiked to 39.4 degrees Celsius, there was an increase in cough and dyspnoea. X-ray of the chest showed an increase in pulmonary infiltrate.

Which of the following is the MOST LIKELY explanation of this clinical picture? a) Normally resolving pneumococcal pneumonia b) Laboratory contamination of original culture c) Superinfection d) Side effect of therapy e) Pulmonary thromboembolism

D. This clinical picture demonstrates deterioration in the patient's condition, and is not part of normally resolving pneumonia. Drug fever, or serum sickness, usually occurs on the 7th to 12th day of antibiotic therapy and can produce unexpected fevers, skin rash and an eosinophilic pulmonary infiltrate. Although more commonly due to penicillins it can be caused by tetracycline. As he had initially improved, it is unlikely that the original specimens were contaminated. Pulmonary embolism does not cause a high fever. Superinfection is commonly due to gram-negative bacteria, fungi or resistant staphylococci and usually appears on the 4th or 5th day. Question 2 A 21 year old man walks into your surgery with his head tilted sideways, his eyes rolled up and his tongue sticking out. He speaks with difficulty but says that he has been 'stuck' in this position since taking a new medicine a few hours ago 'for his nerves'. You should administer: a) Diazepam b) Benztropine c) Chlorpromazine d) Phenytoin e) Haloperidol B. The presentation described is an acute dystonic reaction, which involves a spasmodic torticollis, (where the head is pulled and held to the left or right by one or other sternomastoid), upward drawn eyes and an open mouth (oromandibular dystonia). It may occur (particularly in young men) within a few days of starting a neuroleptic medication. Treatment is with the anticholinergic medication eg. benztropine. The dystonias are a group of disorders involving prolonged spasms of muscle contraction. Spasmodic torticollis is one type, as is trismus (clenched jaw) and Blepharospasm (involuntary contraction of the orbicularis oculi). In isolation, the dystonias are usually of unknown cause and treatment is difficult.

Question 5 You are called to see a 78 year old woman with a three-week history of headaches and depressive symptoms. She relates that 24 hours ago the vision in her right eye suddenly dimmed. Today the visual acuity in the eye is limited to perception of hand movements only, compared with 6/6 in the left eye. She is not known to be a diabetic and her blood pressure is only minimally elevated. Which of the following possible causes of her visual loss require IMMEDIATE investigation and treatment to prevent blindness in the other eye? a) Detached retina b) Central retinal artery occlusion c) Central vein occlusion d) Acute glaucoma e) Temporal arteritis

E. Temporal arteritis is an uncommon disease of the elderly and is characterised by the classic complex of fever, anaemia, high ESR and headaches in an elderly person. It is closely associated with polymyalgia rheumatica. Temporal arteritis is the most common manifestation of a systemic vasculitis. Headache is the predominant symptom and may be associated with a thickened or nodular artery. A serious complication, as described in this lady, is ocular involvement - ischaemic optic neuritis. Most patients have head or eye symptoms for months before objective eye involvement. Acute glaucoma causes a red, painful eye, reduced vision and a fixed, mid- dilated pupil which may be slightly ovoid. The pain may be severe and associated with nausea and vomiting. Acute glaucoma may be preceded by blurred vision or haloes around lights. It is a uniocular attack due to blockage of drainage of aqueous fluid from the anterior chamber via the canal of Schlemm. Urgent treatment with hyperosmotic agents is necessary to reduce the intraocular pressure. Question 6 Which of the following is INCORRECT? a) The incidence of haemochromatosis in Australia is 1:200 b) C282Y homozygotes account for more than 90% of haemochromatosis in Australia c) The majority of patients with one copy each of the C282Y and H63D mutation never develop haemochromatosis

d) 90% of C282Y homozygotes develop symptoms of the disorder at some stage in their lives e) Carriers of one copy of the altered gene are generally healthy D. Studies estimate that up to 50% of C282Y homozygotes will remain symptom free throughout their life.

Question 10 A 21 year old female patient presents with lower abdominal pain and tenderness at 14 weeks of gestation. Her temperature is 38.5 degrees Celsius. The most important diagnosis to EXCLUDE is: a) Pyelonephritis b) Threatened abortion c) Ectopic pregnancy d) Degeneration of a uterine fibroid e) Appendicitis E. Appendicitis is the commonest surgical emergency and has its maximum incidence between 20 and 30 years of age. In pregnancy it occurs mainly in the second trimester. Pain is generally higher and more lateral than typical appendicitis. Ectopic pregnancy occurs approximately one in every 100 clinically recognised pregnancies. The classical triad of ectopic pregnancy includes amenorrhoea (65-80%), lower abdominal pain (95+%) and abnormal vaginal bleeding (65-85%). Degeneration of a uterine fibromyoma typically occurs in the second trimester of pregnancy and is due to ischaemic necrosis. In threatened abortion there is vaginal bleeding. Pain is usually not a significant feature unless the cervix is beginning to open. Pyelonephritis can mimic acute appendicitis in pregnancy. * Question 11 A patient presents with a recurrent severe hemicranial nocturnal headache which lasts for 60 minutes and occurs regularly every 3 weeks. The headache is accompanied by a blocked nose and watering eye. The MOST LIKELY diagnosis is: a) Tension headache b) Migraine c) Sinusitis d) Cluster headache e) Trigeminal neuralgia

D. Cluster headache has a four-fold higher incidence in men than women. It is characterised by constant unilateral orbital pain, with onset usually within 2-3 hours of falling asleep. The pain is intense and steady with lacrimation, blocked nostril then rhinorrhoea and sometimes miosis, ptosis, flush and oedema of the cheek, all lasting approximately an hour or two. It tends to occur nightly for several weeks or a few months, followed by complete freedom for months or even years. The response to inhaled oxygen can be dramatic. Question 17 Sudden onset of unilateral orbital pain, photophobia, lacrimation and blepharospasm suggests a diagnosis of: a) Open-angle glaucoma b) Iritis c) Temporal arteritis d) Blepharitis e) Vitreous haemorrhage B. Acute iritis presents with pain of acute onset, photophobia, blurred vision, lacrimation, circumcorneal redness (ciliary congestion) and a small pupil (initially from iris spasm). Talbot's test is positive: pain increases as the eyes converge (and pupils constrict). The slit lamp reveals white precipitates on the back of the cornea and anterior chamber pus (hypopyon). Open angle glaucoma is painless and largely asymptomatic until there is visual field loss; temporal arteritis causes pain in the temporal area, not in the orbit; blepharitis is inflammation of the eyelids; vitreous haemorrhage can present with visual field loss depending on the size of the haemorrhage and it is generally painless. Question 23 Which finding is INCONSISTENT in this report of a cerebrospinal fluid (C.S.F.) examination? a) Elevated protein b) Normal chloride c) Elevated glucose d) No red cells e) Gross excess of polymorphonuclear leucocytes

C. A lumbar puncture consistent with a diagnosis of pyogenic meningitis contains excessive polymorphs, with protein at 1.5g/l (aseptic less than 1.5g/l) and glucose at less than 2/3 the plasma level. There are no red cells unless it is a bloody tap (ie. Artefact due to piercing blood vessel). Question 32 Jane is brought into the surgery after being struck in the eye with a tennis ball. On examination you note blood in the anterior chamber of the eye. Which of the following statements regarding her management is INCORRECT? a) Analgesia containing aspirin is contraindicated b) Management is directed at avoiding the risk of secondary haemorrhage c) Emetics may be required d) Decreased visual acuity necessitates the exclusion of other ocular damage coexisting with the hyperaemia e) Jane should be referred to an ophthalmologist for urgent review and management C. The management of hyphaema is aimed at preventing secondary haemorrhage which can cause the anterior chamber to be filled with blood and severe secondary glaucoma to develop. Thus aspirin-containing analgesics should be avoided (due to the risk of increased bleeding) and management includes strict bed-rest in hospital. Although reduced visual acuity can occur, it is important to exclude other ocular damage through specialist referral if this occurs. Vomiting needs to be prevented NOT induced, as it raises intraocular pressure and increases the risk of secondary bleeding. * Question 33 The MOST LIKELY venous source of fatal pulmonary embolism is: a) Iliofemoral b) Subclavian c) Saphenous d) Pelvic e) Popliteal A. Most pulmonary emboli arise from proximal deep vein thrombosis (deep veins of lower limb, pelvis and inferior vena cava). Less frequently, thromboses of the upper arm are the source. Saphenous vein thrombosis seldom results in clinically obvious pulmonary embolism. Also, in order for

the thrombus to cause fatality, it has to be large enough to either cause obstruction in the right atrium or of the right ventricular outflow tract. It therefore would have to originate in a large vein.

Question 37 A 40 year old Vietnamese man who arrived in Darwin 6 weeks prior, presents with 5 days of headache, fever and malaise. What is the MOST LIKELY diagnosis? a) Malaria b) Typhoid fever c) Dengue fever d) Filariasis e) Meningococcal meningitis A. Malaria is endemic in South-east Asia and a clinical presentation such as this should be regarded as malaria until proven otherwise. Its symptoms are usually non-specific, with headache, fevers and malaise being the most common symptoms. Typhoid fever has an incubation period of 7 to 21 days, with an average of 14 days. Although headache and fever are common symptoms, one would expect other symptoms such as abdominal pain and diarrhoea. Dengue fever has an incubation period of only 5-8 days, followed by sudden headache, fevers and severe myalgias ("The Dandy Walker Syndrome"). A rash typically develops on the third to fifth day. Filiariasis (due to Wucheria bancrofti) can present as a lymphangitis, with recurrent fever and inflammation overlying the affected lymphatic vessel. Finally meningococcal meningitis has a rapid clinical course and the patient would be unlikely to present with such a long clinical history. Photophobia, neck stiffness and altered conscious state could also be expected. Question 40 The most important muscle used for inspiration is: a) External intercostal muscle b) Diaphragm c) Scalene muscles d) Rectus abdominis e) Internal intercostal muscles

B. In resting healthy individuals, contraction of the diaphragm is responsible for the majority of inspiration. Clinically, it is important to remember that that the diaphragm is innervated by the C3/4/5 spinal segments, (mostly C4/5) via the phrenic nerves. Loss of function of this segment, either from trauma or metastatic malignancy, will result in the loss of the function of the diaphragm and all intercostal muscles. The patient will die of respiratory exhaustion in three days unless respiration is supported. Unilateral phrenic nerve damage (i.e. bronchogenic carcinoma), causes unilateral hemi diaphragmatic paralysis which can cause a 20% loss of inspiratory effort. This is, remarkably, quite asymptomatic. Question 1 Intestinal sugar malabsorption following gastroenteritis may be suspected if the child has: a) Vomiting each time the particular sugar is eaten b) A stool pH of 6 c) Salmonella in the faeces d) Glycosuria and excess sugar in the faeces e) Recurrence of diarrhoea E. Sugar tolerance (usually lactose) is recognised by the reappearance of diarrhoea when milk is reintroduced to the diet. It occurs more often in the younger infant. The stools become watery and frothy and tend to excoriate the buttocks. Fluid stools may be positive for reducing sugars such as such as lactose/glucose but not sucrose. Sugar intolerance is not associated with any particular pathogen. Approximately 50% of infants under 6 months who have required hospitalisation for gastroenteritis require a low lactose formula at the time of discharge. * Question 2 Following appendicectomy 10 days previously, a patient presents with fever and mucus diarrhoea. The MOST LIKELY cause is: a) Viral gastroenteritis b) Pelvic abscess c) Staphylococcal enteritis d) Giardiasis e) Abscess around appendiceal stump

B. Passage of mucus with diarrhoeal stool and fever in a patient recovering from peritonitis (appendicitis being the most common cause), is PATHOGNOMIC of pelvic abscess. It should be therefore be thought of first and either treated or excluded. Viral gastroenteritis causes a fluid, not mucus diarrhoea. Staphyloccoccal enteritis would be a fulminating, severe illness with more worrying signs than just change in stool. Giardia causes a fatty, large, pale, offensive stool with undigested vegetable material (peas, carrots and corn) in an otherwise well individual. An abscess around the appendiceal stump would cause signs of a localised peritonitis. Question 4 An 5 month old infant is brought to the emergency department with a few hours history of intermittent screaming and drawing up his knees. He has vomited twice. He is pale, clammy and tired. He also started vomiting. No abnormality is found on abdominal examination. What is the MOST LIKELY diagnosis? a) Gastro-enteritis b) Intussusception c) Urinary tract infection (UTI) d) Hirschsprung's disease e) Pyloric Stenosis B. Intussusception occurs when one segment of the bowel passes inwards inside the adjacent distal bowel. It is the most common cause of intestinal obstruction between 3 months old to 6 years old (rarely after 36 months old). The peak incidence in infants is 4-7 months. In typical cases there is sudden onset, in a previously well child, of severe paroxysmal pain which recurs at frequent intervals and is accompanied by straining effects and loud cries. Vomiting usually occurs once or twice in the first hours and then reappears once obstruction is established. About half will pass red currant jelly stools. Initially, physical examination may be normal, but a sausage-shaped mass in the right upper abdomen is found in more than half of infants in the first few hours. Later on abdominal distension and increased tenderness may conceal the mass. Hirschprung's disease (aganglionic megacolon) usually presents in the neonate but at a later age tends to present as chronic constipation with overflow soiling. Pyloric stenosis usually commences between 3 to 6 weeks of age and is rare in infants older than 11 weeks. It is associated with projectile vomiting. Question 10

In the newborn physiological jaundice is a diagnosis of exclusion. Which of the following factors is CONSISTENT with a diagnosis of physiological jaundice? a) Onset within the first 24 hours b) Unwell infant c) Jaundice improves with increased feeds and hydration d) Preterm infant e) Jaundice lasting longer than 2 weeks

C. Physiological jaundice is often aggravated by deficient feeding and so increased feeds/fluids is often all that is necessary to improve physiological jaundice. Factors suggesting that a pathological cause for the jaundice should be actively sought include: jaundice within the first 24 hours; jaundice persisting 7-10 days in a full-term infant; the sick infant; high levels of serum bilirubin. More serious causes of jaundice include gram negative septicaemia, biliary atresia or Rh incompatibility. * Question 11 A 65-year-old woman presents with fluid abdominal enlargement to the xiphisternum. On clinical examination this is thought to be due to an ovarian cyst. The MOST APPROPRIATE next step in management is: a) Laparotomy b) Paracentesis to obtain cells for cytology c) Laparoscopy d) Cytotoxic drug therapy e) Plain x ray of the abdomen A. As the clinical diagnosis infers a risk of malignancy, diagnosis should be established by laparotomy with full removal of the mass if possible. Cytotoxic drug therapy may only be indicated after histological diagnosis and staging are confirmed. Paracentesis may not give a histological diagnosis and may spill malignant cells into the peritoneum. Laparoscopy would be of no diagnostic or therapeutic benefit, and may be procedurally impossible. Plain X ray of the abdomen is of very limited use, and may only be of help in the case of a teratoma.

Question 14

Mrs Jones suffers with rheumatoid arthritis which is at present well controlled. She is concerned about a rash on her hands which she has only recently noticed. It is itchy. Clinically you suspect that this is lichen planus. Which of the following is INCORRECT? a) Lichen planus commonly occurs on the wrists, shins and lower back b) The rash could have been precipitated by recent gold injections c) A lichenoid eruption begins within a few days of commencing the causative medication d) This rash could have been precipitated by non steroidal antiinflammatory drugs (NSAIDs) e) This rash could be related to blood pressure medication C. Lichenoid drug eruptions may occur after drug therapy has commenced but it takes months not days and takes some time to resolve. Gold, NSAIDS, antimalarials, the antihypertensive captopril and thiazide diuretics can precipitate the reaction.

Question 19 The MOST COMMON cause of visible bright blood on a child's stool is: a) Colitis b) Rectal polyp c) Intussusception d) Fissure-in-ano e) Peptic ulcer D. In an otherwise well child, the commonest cause of visible bright blood on a child's stool is fissure-in-ano usually due to the passage of a large motion. The fissure can be seen by gently parting the anus. Pain is a distinguishing feature. The commonest cause of blood mixed with stool in an unwell child is gastroenteritis caused by pathogenic bacteria. Non-infectious inflammatory colitis is rare in childhood, as are rectal polyps. The clinical signs of intussusception are of a sudden onset of severe colicky abdominal pain in a child most commonly between 3 months and 7 months of age. In about 50% there is passage of red currant jelly stools. Peptic ulcer disease is very unusual in childhood. * Question 20 The BEST SIMPLE estimate of the extent of blood loss in a patient who has recently suffered a large upper gastrointestinal bleed is:

a) Volume of loss by history b) Observation of the amount of melena c) Monitoring of the pulse rate d) Hypotension when the patient sits up e) The admission of haematocrit D. blood loss is haemodynamically significant (>500ml), the patient may feel faint and show pallor. There will be reduced circulating blood volume, which will be evident as postural hypotension. Volume loss cannot be estimated by history or by observation; melena may not occur for hours after the bleed. Tachycardia may be due to, or exacerbated by, anxiety and pain; haematocrit may not change after a sudden bleed until haemodilution occurs. In larger centres, central venous pressure monitoring would be instituted. Question 24 The daily fluid requirement of a healthy 4.5 kg (4500 g) 2 month-old infant in a temperate climate is: a) 400 ml b) 500 ml c) 675 ml d) 1250 ml e) 1500 ml C. Oral fluid requirements for infants 0-1 year old is 150ml/kg/day . Question 32 The INITIAL management of a spontaneous pneumothorax in a patient without dyspnoea at rest and a uniform 2 cm separation of the lung from the chest wall on a radiograph should consist of: a) Observation b) Single needle aspiration c) Positive pressure respiration d) Underwater drainage e) Underwater drainage with suction A. A small pneumothorax can be managed with observation and avoidance of situations likely to reduce atmospheric pressure (i.e. flying). Where the lung is halfway to the heart border or more, or the pneumothorax occupies 30% of

the pleural space, aspiration with or without drainage by intercostal catheterisation is indicated. Flying is avoided because the reduction in extrathoracic pressure would make the pneumothorax worse. Question 34 A 20 year old man was admitted to hospital because of the sudden onset of intense headache and neck stiffness. For the past 4 years, the patient had had occasional focal motor seizures on the right side of the body. Which of the following is the MOST likely diagnosis? a) Ruptured saccular (berry) aneurysm b) Hypertensive cerebral haemorrhage c) Haemorrhagic infarction d) Haemorrhage in cerebral neoplasm e) Arteriovenous malformation of brain E. This young man presents with symptoms consistent with an acute subarachnoid haemorrhage. 80% of subarachnoid haemorrhages are caused by ruptured aneurysms (usually congenital Berry aneurysms), with the second biggest cause (5%) being AV malformations. 85% of Berry aneurysms occur around the Circle of Willis with possible warning signs being 3rd or 6th nerve palsies, pain behind the eye and other symptoms consistent with an expanding mass in this area. AV malformations are present from birth but usually present with symptoms between 10 and 30 years of age. Chief clinical symptoms and signs are headache, seizure and those associated with rupture. Focal seizures that become generalised occur in 30% of cases. The past history of focal motor seizures would suggest the presence of an arteriovenous malformation over the other causes mentioned. Question 4 Eve is 9 weeks pregnant. She presents with a history of 24 hours of intermittent dark blood loss staining her underwear. On vaginal examination, the uterus is the expected size and the cervical os is closed and non-tender. You should advise her that: a) she is likely to have miscarried b) approximately half of all pregnant women suffer bleeding during the first trimester c) she requires admission to hospital for a D&C d) there is a higher incidence of congenital malformations in fetuses where bleeding occurred in the first trimester e) she should have an ultrasound

E. Eve has a threatened abortion as the os is closed and there is no history of passage of products of conception. This occurs in 25% of pregnancies and half of these progress to spontaneous abortion. For those who carry the pregnancy to term there is however no increased risk of congenital malformation. An ultrasound examination will be helpful, as it will demonstrate the presence (or absence) of a foetus appropriately sized for dates. The demonstration of a foetal heart is very reassuring. At 9 weeks, when a foetal heartbeat is present and not slow, 90% of pregnancies will continue despite bleeding. Question 6 Vera Smith, aged 61, has noticed a swelling in the front of her neck, which is enlarging quite rapidly. She is otherwise well, without symptoms of hypothyroidism or hyperthyroidism. On examination she has a multinodular goitre, with a dominant nodule of 4 cm diameter in the right upper pole of her thyroid gland. Of the following investigations, which is the MOST important? a) TSH, T3, T4 b) ultrasound Scan c) fine Needle Biopsy of dominant nodule d) CT scan e) nuclear Medicine Scan

C.The incidence of malignancy in a dominant thyroid nodule is approximately 7%, and the fine needle biopsy is the appropriate investigation to exclude malignancy. Vera has several features consistent with a thyroid malignancy, including a rapidly growing solitary lump, her age (over 60) and the fact that the dominant nodule is >3cm. Detection or exclusion of malignancy is the MOST important issue initially. Thyroid function tests must also be performed routinely even though the patient is clinically euthyroid but they do not help in determining malignancy. Ultrasound scan adds little to clinical examination, but may be used to guide the biopsy needle. CT scan may be useful to assess retrosternal extension, but there are currently no symptoms of this problem. Nuclear medicine scan is useful to determine the functional status of a nodule but carcinoma cannot be excluded on the basis of radionuclide scan. Question 10 A 26-year-old G3P2 who is 30 weeks gestation telephones you to say that she has had an episode of bright red vaginal bleeding. It occurred several hours ago and she has had no associated symptoms. She estimates she lost about a

teaspoon of blood. What would be the MOST APPROPRIATE IMMEDIATE management? a) tell her to call back if it happens again b) tell her to come in immediately for further evaluation c) make her an appointment at your next antenatal clinic d) send her directly for an ultrasound examination e) ask her to monitor fetal movements for the next 24 hours and to ring you tomorrow C. Although this bleeding may have a benign cause it is important to recognise that she may have a placenta praevia or have had a placental abruption. Both of these conditions require urgent attention because of the risk of maternal and fetal morbidity and mortality. The patient should be examined (vital signs, examination of the uterus and foetus and a sterile speculum examination to assess the nature of the bleeding and whether or not the os is closed). An ultrasound to assist in confirmation of the diagnosis will be necessary as part of your evaluation but should occur only after the patient has been assessed and is stable. Question 15 Michael Peters, aged 12, sustained a blow to his left temple when he fell while climbing a tree. He was dazed, but able to recount what had happened. An hour later he complained of an increasingly severe headache and vomited once, and then was brought to hospital. His pulse is now 54 bpm, BP 130/90 mm Hg and he is drowsy and confused. His left pupil is larger than his right. Which is the MOST appropriate advice to give Michael's parents? a) Michael probably has a skull fracture, and will need an urgent skull x ray b) Michael has severe concussion, and will be observed closely overnight c) Michael requires emergency surgery as soon as possible d) Michael requires an urgent CT scan of his head as he may have bleeding into his brain e) Michael is gravely ill and has only a 50% chance of survival of bleed C. The history of the injury is highly suggestive of an extradural (epidural) haematoma. Although Michael did not lose consciousness initially, his condition has deteriorated rapidly and significantly. He is now bradycardic, hypertensive and his pupil is dilated on the side of the injury. His level of consciousness is also deteriorating. An extradural haematoma will result in death if not evacuated promptly. There is approximately 75% chance he will have a fracture overlying the haematoma, but skull x ray is not indicated. There may be 'concussive' injury

to the underlying brain but the life-threatening factor is the raised intracranial pressure from the extradural haematoma.While it would be ideal to have a CT scan, the rapidly deteriorating condition means that emergency surgery should not be delayed. Michael is clearly gravely ill, but the mortality from SAH is around 10% for obtunded patients and 40% for patients who are comatose prior to surgery. Prognosis is better for young patients, but deteriorates with associated other intracranial injuries and with delay between injury and surgical intervention. Question 26 Felicia is a 15 month old girl. She has symptoms and signs consistent with a viral upper respiratory tract infection (URTI), including a fever of 38.8 degrees Celsius. Her weight is 11kg. Which of the following is CORRECT regarding the prescription of oral paracetamol in this case? a) daily dose should not exceed 90mg/kg/day b) dosage should be calculated at 30mg/kg/dose 4 hourly c) dosage should be calculated at 15mg/kg/dose 6 hourly d) dosage should be calculated at 20mg/kg/dose 6 hourly e) daily dosage should not exceed 60mg/kg/day A. Daily oral paracetamol dosage should not exceed 90mg/kg/day, up to a maximum of 4g. 60mg/kg/day is the maximum dosage for infants aged less than 6 months. The recommended paracetamol dose in children is 15mg/kg orally every 4 hours, or 20mg/kg rectally every 6 hours. Question 33 When treating a premenopausal woman who has irregular cycles, severe hot flushes and no contraindications to hormone replacement therapy the MOST APPROPRIATE therapy is: a) continuous oestrogen therapy b) continuous combined (oestrogen and progestogen) therapy c) sequential oestrogen therapy d) sequential combined (oestrogen and progestogen) therapy e) continuous progestogen therapy D. Hormone replacement therapy is indicated in women who are suffering from severe menopausal symptoms provided they have no contraindications to its use.

Sequential combined HRT is the best option for premenopausal women who do not require contraception as it can alleviate symptoms and control irregular cycles. The use of continuous or sequential unopposed oestrogen is associated with endometrial hyperplasia and the development of endometrial cancer and is contraindicated in women who have not had a hysterectomy. Continuous combined HRT is recommended for the treatment of symptoms in women who are more than one year postmenopausal. These women will probably remain amenorrhoeic on such a regimen. Progestogen alone is not always effective at treating menopausal symptoms. Question 35 Teresa, 25 years, presents having experienced an episode of postcoital bleeding two days ago. What is the MOST APPROPRIATE management? a) reassure her and ask her to return if it recurs b) undertake cauterisation of the cervix to prevent further bleeding c) treat her with metronidazole gel to eradicate infection d) send her to the emergency department for immediate assessment e) undertake diagnostic cervical cytology and screening for sexually transmitted diseases

E. Postcoital bleeding is a serious symptom that could be indicative of cervical pathology. It is not an emergency requiring assessment in hospital. Common causes of postcoital bleeding include a cervical erosion, an infection such as chlamydia and other less common pathologies in this age group such as a cervical polyp. Medical practitioners must however ensure that they exclude precancerous or cancerous lesions of the cervix by making sure that cervical cytology (Pap smear) is performed as well as appropriate STI (sexually transmitted infection) screening. If the bleeding is recurrent, or the cervix looks abnormal colposcopy is recommended. Cauterisation of the cervix is sometimes performed if a friable cervical erosion is present, bleeding is recurrent and other cervical pathology has been excluded. Question 39 Angelina, 27 years G3 P2, has a transverse lie at 36 weeks gestation. Should her membranes rupture what would be the MOST APPROPRIATE MANAGEMENT? a) advise her to come in to labour ward when contractions are five minutely b) lie her down on her side and take her straight to the operating theatre for a caesarean section

c) wait for the head to engage with contractions and proceed to a normal vaginal delivery d) commence intravenous augmentation of labour in order to facilitate a swift delivery e) instruct the patient to adopt the knee chest position (kneeling with head down) and transfer her to theatre for an immediate caesarean section E. Cord prolapse occurs when the umbilical cord lies beside or in front of the presenting part. It is more common in malpresentations, polyhydramnios, during breech deliveries and with premature rupture of the membranes. It is an obstetric emergency, as the umbilical vessels constrict once exposed to the extrauterine environment. Unless the cervix is fully dilated and an immediate operative vaginal delivery can be conducted, an emergency caesarean section is required. During the transfer to theatre the woman should be positioned so that gravity can assist in keeping the presenting part off the cord, i.e the knee - chest position. The presenting part should also be pushed up and away from the cord digitally in order to reduce pressure on the cord. Question 5 Wasting of the thenar muscles as an isolated sign is MOST likely due to: a) Syringomyelia b) Median nerve compression at the wrist c) Motor neurone disease d) Ulnar nerve lesion e) C7-T1 lesion BBB Question 9 Sue aged 3 years is having frequent episodes of wheeze, despite daily inhaled sodium cromoglycate, and regular inhaled salbutamol. What would be the next MOST appropriate step in establishing satisfactory control of Sue's asthma? a) Prescribe inhaled corticosteroid b) Increase sodium cromoglycate dose c) Review the diagnosis d) Substitute large volume spacer e) Check compliance AAA Question 12

Mary, aged 55 years, presents with stiffness of her hips and shoulders, low grade fever, headaches and malaise. There is no objective evidence of arthritis. Her erythrocyte sedimentation rate is 65 mm/hr. The MOST LIKELY cause of this presentation is: a) Subacute thyroiditis b) Polymyalgia rheumatica c) Rheumatoid arthritis d) Influenza e) Rheumatic fever BBB Question 14 Jeremy is 45 years of age and hobbles into your surgery with a swollen knee. He states that it has slowly become worse during the past 3 days. He can't remember any trauma. He is in a lot of pain with an obvious joint effusion. The MOST IMPORTANT diagnostic procedure to undertake is: a) X-ray of knee b) Serum uric acid level c) Aspiration of the joint d) Full blood count e) Urinary uric acid CCC Question 22 June, aged 69 years, is brought to see you by her daughter who is concerned that she has been increasingly forgetful over recent months, and appears to be losing interest in her usual activities. She is also agitated at times. As part of your assessment, you perform a Mini Mental State examination and diagnose dementia. All of the following are reversible causes of dementia EXCEPT: a) Hypothyroidism b) Alcohol abuse c) Thiamine deficiency d) Vitamin B12 deficiency e) Vascular dementia EEE Question 31 Upper motor neurone lesions are characterised by all of the following EXCEPT: a) Loss of voluntary movements b) Increased stretch reflexes

c) Increased muscle tone d) Rapid wasting of affected muscles e) Extensor plantar responses DDD *Question 36 Bob, aged 7 years, is brought to see you because he is complaining of colicky abdominal pains. He has a rash on the back of his legs, buttocks and extensor surface of his forearms. Urinalysis reveals proteinuria and haematuria. The MOST LIKELY diagnosis is: a) Systemic lupus erythematosus b) Anaphylactoid purpura (Henoch Schonlein purpura) c) Post-streptococcal glomerulonephritis d) Polyarteritis nodosa e) Dermatomyositis BBB Question 41 A man presents unwell to Casualty and the following results are obtained from pathology: Urea 15 mmol/L (N 2.0 - 8.5); glucose 5.0 mmol/L (3.5-7.9); Bicarbonate 39 mmol/L (N 20-29); sodium 140mmol/L (135-145); serum potassium 2.4 mmol/L (N 3.5 - 5.0). The most likely cause of these disturbances in your patient is: a) b) c) d) e)

Acute nephritis Severe vomiting Dehydration Diabetes mellitus Chronic renal failure

BBB * Question 42 Which of the following can result from a snake bite? a) Coagulopathy b) Rhabdomyolysis c) Neurotoxicity d) Hyperkalaemia e) All of the above EEE

Question 46 Prolonged, persistent bradyarrhythmia associated with atrio-ventricular heart block is BEST treated with which of the following? a) An artificial pacemaker b) Isoprenaline hydrochloride c) Disopyramide d) Cisapride e) All of the above AAA Question 51 The risk of a baby dying of SIDS is reduced by all of the following EXCEPT: a) Removing soft toys from the cot b) Dressing the baby appropriately for the weather c) Lying the baby prone d) Not using baby pillows e) Apnoea alarms CCC Question 55 A 70 year old presents with acute central and peri-umbilical abdominal pain which gradually increases in intensity. She is vomiting profusely and develops watery diarrhoea with flecks of blood. Examination of the abdomen reveals localised periumbilical tenderness with some rigidity. Rectal examination is normal. An irregular pulse is noted and an ECG reveals Atrial Fibrillation. Optimal management should be: a) Nasogastric suction and intravenous fluids b) Intramuscular Pethidine and review c) Intravenous Cephalosporin d) Intravenous Somatostatin e) Surgical intervention EEE Question 56 In making a diagnosis of a transmural myocardial infarction from an electrocardiographic tracing, the MOST SIGNIFICANT finding is: a) Prolongation of the PR interval b) The presence of pathological Q waves c) A raised ST segment in V leads d) Inverted T waves e) Depression of the ST segment in affected leads

BBB * Question 57 All the following statements concerning Paget's disease of the nipple are correct EXCEPT:a) The basic lesion is an intra-ductal carcinoma b) The nipple epithelium is infiltrated with cancer cells c) The tumour can always be detected on careful clinical examination d) The first symptom is often itching or burning of the nipple e) The diagnosis is readily established by biopsy of the erosion CCC Question 59 Which of the following is the most useful observation to EXCLUDE the diagnosis of fracture of the neck of the femur? a) Normal buttock contour b) Quadriceps tone c) Absence of local tenderness of the hip joint d) Absence of crepitus of the hip e) Normal range of movement in the hip EEE * Question 60 Which of the following statements is MOST characteristic of mid-trimester bleeding? a) It is of little consequence b) It is related to early effacement and tearing of small vessels c) Praevia or abruption will be found in 25% of cases d) There is a risk of concealed bleeding e) None of the above DDD Question 69 Katelyn Norris, aged 20, has returned for the results of her Pap smear. This is reported as CIN 2. The MOST APPROPRIATE management is: a) Repeat smear after treating infection b) Repeat smear after 3 months c) Cryotherapy to the cervix d) Colposcopy and biopsy e) Cone biopsy DDD * Question 70 Which is the BEST indication of successful treatment of the nephrotic syndrome?

a) b) c) d) e)

Loss of oedema A normal serum creatinine Absence of urinary protein Massive diuresis Normal blood pressure

CCC Question 72 A 32 week pregnant woman comes for the first time with BP of 180/125, proteinuria > 8g, with no oedema. All of the following are true EXCEPT: a) MgSO4 infusion should be started prophylactically b) Hydralazine, either IV or orally, should be given c) She should be delivered by LSCS within 48 hrs d) Betamethasone should be given. e) Continuous CTG monitoring is imperative CCC Question 76 What is the MOST EFFECTIVE treatment for a bleeding cavernous haemangioma in a 2-month-old infant? a) b) c) d) e)

Injection of sclerosant Excision Radiotherapy Diathermy Pressure dressing

EEE Question 79 Which of the following effectively distinguishes delirium from other psychological disturbances? a) Memory impairment b) Hallucinations c) Thought disorder d) Clouded consciousness e) Severe anxiety

DDD

Question 83 Which of the following is CORRECT in relation to severe reflux in infants less than 12 months old? a) Infants have reflux only after feeds b) The oesophagus of normal infants is not exposed to gastric acid c) Irritability caused by reflux is more common in infants under 3 months of age d) Nutrient loss may be present although weight loss is uncommon e) Failure to thrive does not occur DDD *Question 85 Lyn is worried about her 12 month old child as it sometimes appears that her eye is turned. Which of the following statements is INCORRECT? a) Amblyopia(dimness of sight) is generally preventable if strabismus is diagnosed and treated early b) The standard period of eye patching required to reverse amblyopia is 4 weeks c) Strabismus is more common in the premature infant d) Assessment of the symmetry of the infant corneal light reflex may detect squint e) The cover test may be necessary to elicit squint BBB * Question 86 A 5 yr old child presented with acute onset of three weeks' duration of vomiting in the morning associated with headache. His father is concerned that he has been lacking in energy lately. The MOST LIKELY cause for his symptoms is: a) Glioblastoma b) Astrocytoma c) Medulloblastoma d) Migraine e) Meningioma CCC Question 90 Concerning Q Fever (Coxiella burnetti), which of the following is TRUE? a) Only abattoir floor workers are at risk of infection b) The rash is pathognomonic

c) It is spread through contaminated cuts d) It is easily treated with sulphonamides e) Persistent infection can result in endocarditis EEE * Question 91 Mrs Yoshida presents with her 4 year old son, Takahiro. She is concerned that he has had consistently pale malodorous greasy bulky stools and suffers painful abdominal bloating for some weeks now. She says he has become irritable. He was breast-fed up until 10 weeks of age and then transferred to formula and cereals, followed by a traditional Japanese diet. He was quite well until they came to live in Australia one month ago. Clinically, he is afebrile with no abdominal abnormalities. The MOST LIKELY cause for his diarrhoea is: a) Coeliac disease from wheat-derived cereal b) Giardia c) Blastocystis hominis d) Milk allergy e) Maternal/child anxiety due to environmental upheaval AAA * Question 92 A patient slips on a wet floor at work and falls heavily on his buttocks. He complains of back pain, but is able to walk. His only complaint is that his penis and scrotum feel numb. The MOST APPROPRIATE step in further assessment is: a) X-ray of the lumbar spine b) Test power of lower limbs c) Rectal examination d) Quietly investigate the possibility of malingering e) CT lumbar spine. CCC * Question 93 A 5 month old boy presents with his parents. They are concerned that he may have a "lazy eye". The confrontation test was positive. The MOST APPROPRIATE management would be: a) Reassure the parents that the condition is self-limiting b) Refer for CT of the head

c) Instruct parents in a set of exercises to be done daily to strengthen the bad eye d) Patch the good eye e) Refer to an Ophthalmologist EEE Question 95 Which of the following conditions may have a fatal outcome? a) Angry looking napkin rash b) Blue spots on the buttocks c) Blue spots on the buccal mucosa d) Diamond shaped area of hair loss on the scalp e) Variable-aged bruises with demarcated edges EEE * Question 96 Regarding sciatic pain, which of the following is TRUE? a) It is only felt in the buttock b) It can occur in the absence of disc prolapse c) The pain is referred to the anterior thigh d) It is usually cured by operation e) Groin pain is highly suggestive of nerve entrapment EEE Question 100 A three year old child has a generalized seizure lasting 3 minutes. He has a temperature of 38.5 degrees Celsius. After 30 minutes, he is seen to be playing happily with his mother. The MOST APPROPRIATE next step in management is: a) Arrange outpatient neurological review with EEG b) Perform lumbar puncture c) Arrange for FBE, blood cultures and chest X-ray d) Commence valproate prophylactically e) None of the above EEE Question 104 Terri is 16 years old and wishes to take the contraceptive pill. The BEST clinical predictor of post-pill amenorrhoea is:

a) b) c) d) e)

Oestrogen dose in the pill Age since menarche Use of sequential formulations Duration of regular cycling Serum FSH and oestradiol mid-cycle

DDD * Question 105 The MOST COMMON cause of chronic pancreatitis is: a) Virus infection b) Gallstones c) Alcohol d) Carcinoma e) Illicit IV drug use CCC * Question 106 Jenny is three years old. At this age she should be able to: a) Ride a three-wheeler bike b) Create a tower of 8 cubes c) Do up her own buttons d) Name four colours e) Tie shoe laces AAA * Question 107 Elsie Warne, an 82 year old woman, has been a Nursing Home resident since suffering a Right parietal CVA three years prior. On presentation, she is obese and appears demented. Her BP is 160/100, pulse 60regular, and she has macroglossia and coarse skin. Her cholesterol is 12.4mmol/L. Her sudden death, three weeks later, was MOST LIKELY caused by: a) Complete heart block b) Hypertension induced Cerebrovascular accident c) Excessive thyroxine replacement d) Acute myocardial infarction e) Chronic obstructive sleep apnoea syndrome CCC Question 110

In children, an innocent cardiac murmur has all of the following attributes EXCEPT: a) Variation in loudness with change in posture b) Murmur heard in early diastole c) An ejection click d) Absence of a thrill e) Continuous murmur through systole and diastole CCC Question 112 A Pap report reads "No cervical columnar cells seen. No cellular abnormalities seen. Red blood cells and bacteria present. Inflammatory-type smear". This result is not reliable because: a) The transitional layer was not sampled b) Inflammation was present c) The smear was traumatic d) A spatula was used instead of a cytobrush e) It is clinically irrelevant AAA * Question 113 Concerning children with asthma, all of the following are recommended EXCEPT: a) Education of the parents b) Development of asthma care plans c) Ready use of oral prednisolone d) Access to "reliever" at the child's discretion e) Desensitization to allergens orally or parenterally EEE * Question 114 Heidi Brown, a 22 year old teacher, presented complaining of increasing dysmenorrhoea and deep dyspareunia. Clinically, the speculum exam was normal, but there was a feeling of discomfort on the left with bimanual examination. Heidi's problem is MOST LIKELY due to: a) Pelvic inflammatory disease b) Pre-menstrual syndrome c) Psychological problems d) Endometriosis e) Fibroid necrosis

DDD Question 116 Mr John Hill is a 25 year-old optometrist with no previous history of health problems and no significant family history. He presents with a 1 month history of alternating bowel habit, with bloating and pain, flatulence, abdominal distension, the passage of mucus without blood rectally and a feeling of incomplete evacuation. He gains relief from his symptoms with defecation and it never troubles him at night. He has not lost any weight and his appetite is unchanged. He finds that his abdominal discomfort is interfering with his work. He has recently taken on a large debt. Which of the following is CORRECT? a) Colonoscopy is indicated to exclude malignancy b) He should be managed with reassurance and education c) Small bowel biopsy is indicated to exclude Crohn's disease d) A full blood screen, including CEA and HIV serology is required e) His symptoms will settle after the debt is under control BBB * Question 117 The patient above presented feeling unwell with a mild fever. The rash had appeared the day before. The condition is:

a) b) c) d) e)

Erythema marginatum Erythema infectiosum Erythema nodosum Erythema multiforme Erythema ab igne

DDD Question 120

Signs of left ventricular failure consist of all of the following EXCEPT: a) Gallop rhythm b) Low volume pulse c) Pedal oedema d) Basal crepitations e) Paroxysmal nocturnal dyspnoea

CCC Question 124 This lesion has been present at the root of this patient's neck for ten months. Its appearance is most consistent with a:

a) b) c) d) e)

Nodular melanoma Pigmented basal cell carcinoma Seborrheic keratosis Hutchinson's melanotic freckle Squamous cell carcinoma

BBB * Question 125 Jordan Miller, aged 19, is bought to Emergency after fainting while running a 100 metre sprint at his Athletics Club. He is extremely fit, and exercises frequently. He states that it was the second time he "blacked out" while running. There are no other symptoms except a 2/6 systolic bruit along the left sternal edge best heard with standing. Which of the following is the MOST LIKELY diagnosis? a) Hypertrophic obstructive cardiomyopathy b) Aortic stenosis c) Atrial myxoma d) Prolapsing mitral valve

e) Re-entrant tachycardia AAA Question 43 Peta, aged 3, presents distressed with an acute attack of asthma. She is treated with salbutamol via MDA and face-mask. She improves symptomatically, but becomes breathless and wheezy again after 90 minutes. The next MOST APPROPRIATE step in management would be:a) Double the dose of Salbutamol b) Repeat Salbutamol and commence oral steroids c) Assess with spirometry d) Measure Oxygen saturation to determine need for hospitalization e) Commence I.V. Aminophylline BBB Question 46 A 58 year old lady presents with painless jaundice. Physical examination of the abdomen is unremarkable. The results of her blood tests are: Liver function tests: Bilirubin 90 umol/L (
View more...

Comments

Copyright ©2017 KUPDF Inc.
SUPPORT KUPDF